11
$\begingroup$

Given a group $G$, the subgroup growth functions are given by

$a_n(G) :=$ the number of subgroups of $G$ of index exactly $n$.

$s_n(G) :=$ the number of subgroups of $G$ of index at most $n$.

The latter is clearly increasing. I wonder when the sequence $a_n(G)$ enjoys similar behavior.

To be specific, say a sequence $\{ x_n \}_{n=1}^\infty$ is roughly increasing if there exists $C > 1$ such that $x_{C n} \geq x_n$ for all natural numbers $n$.

Question: For what groups $G$ is $a_n(G)$ roughly increasing? In particular, is $a_n(G)$ roughly increasing for $G = SL_3(\mathbb{Z})$?

Note that the sequence $a_n(G)$ is roughly increasing for any finitely generated nilpotent $G$ that is infinite. On the other hand, the sequence $a_n(G)$ is never roughly increasing for finite $G$.

$\endgroup$
8
  • $\begingroup$ Does this not follow from the rather precise estimates for $a_n$ obtained by Lubotzky--Nikolov (mathscinet.ams.org/mathscinet-getitem?mr=2155033) for arithmetic groups with CSP? (I'm asking before doing the computation to see if you tried yourself). $\endgroup$ Aug 27, 2018 at 8:55
  • $\begingroup$ Thank you for your reply and for thinking about my question. The main challenge to applying their work is that they only control the growth of $\log(s_n)$. $\endgroup$ Aug 27, 2018 at 10:30
  • $\begingroup$ You're right, I assumed they dealt with $a_n$ but I was mistaken. Do you know if their proof or that of Goldfeld--Pyber--Lubotzky at least implies that $a_n \ge 1$ for sufficiently large $n$? $\endgroup$ Aug 27, 2018 at 11:31
  • 2
    $\begingroup$ I believe the answer to Jean Raimbault's question is no, since there seems to be infinitely many $n$ where $a_n(G) = 0$ for $G = SL_3(\mathbb{Z})$. For this, apply Guralnick's theorem from his J. Algebra paper "Subgroups of Prime Power Index in a Simple Group" to show that for sufficiently large p and k, $a_{p^k}(G) = 0$. Note this doesn't show that $a_n(G)$ is not roughly increasing. $\endgroup$ Aug 27, 2018 at 12:44
  • 1
    $\begingroup$ My gut feeling is that $a_n(SL_3(\mathbb{Z}))$ ought to be roughly increasing. If one takes $C$ to be an index in which a lot of finite-index subgroups of index $C$ in $SL_3(\mathbb{Z})$ exist, then intersecting these with subgroups of index $n$, I would expect "most" to have maximal index, i.e. $Cn$. $\endgroup$
    – Ian Agol
    Aug 29, 2018 at 5:51

0

Your Answer

By clicking “Post Your Answer”, you agree to our terms of service and acknowledge that you have read and understand our privacy policy and code of conduct.

Browse other questions tagged or ask your own question.